You are on page 1of 16

1. A 60-year-old Caucasian man has come to your clinic.

A 24 hour blood pressure monitor shows that he has persistently elevated blood pressure readings
despite having been started on amlodipine 10 mg once a day. He is asymptomatic and not on any
other medication.
What is the best choice of medication now?
(Please select 1 option)
A short acting beta blocker Incorrect answer selected
A thiazide diuretic
An alpha blocking agent
An angiotensin-converting enzyme inhibitor This is the correct answer
Hydralazine
An angiotension-converting enzyme inhibitor and calcium channel blocker such as amlodipine
are first line agents in patients with hypertension without a history of ischaemic heart disease.
2. A 45-year-old man with no past medical history attends the emergency department with an
episode of acute onset breathlessness and chest pain. While in the toilet in the emergency
department he collapses and is found unconscious.
He is moved immediately to the resuscitation area and advanced adult life support is started.
There is no respiratory effort and the monitor shows a sinus tachycardia with no pulse palpable.
His wife attends and informs you he has recently returned from a business trip in Hong Kong and
had been complaining of a painful, swollen left leg. After 10 minutes of unsuccessful advanced
life support, the patient remains in pulseless electrical activity (PEA).
For this cardiac arrest scenario, which is an appropriate next step?
(Please select 1 option)
A FAST scan Incorrect answer selected
B Intravenous thrombolysis followed by CPR for 90 minutes This is the correct answer

C Intravenous thrombolysis followed by CPR for 30 minutes


D Intra-arterial thrombolysis
E Transthoracic echocardiography to image the right ventricle
Thromboembolic disease is a potentially reversible cause of cardiac arrest.
Massive PE can lead to PEA and can be treated with thrombolysis. Thrombolysis is indicated in
the cardiac arrest situation for suspected PEs. However, it can take 90 minutes to be effective and
therefore must only be used if it is appropriate to continue CPR for this duration.
Brief echo (less than 10 seconds) is appropriate in cardiac arrest, but it is unlikely right
ventricular abnormalities (which may indirectly support the diagnosis) can be demonstrated in
that brief time.
FAST scan is not specifically indicated in this scenario.
Intra-arterial thrombolysis is not indicated
3. An 83-year-old previously fit and well male patient is brought to the emergency department
after feeling unwell for some time. While in the waiting room he collapses to the floor and is
unresponsive.
He is rushed to the resuscitation area and attached to a defibrillator. IV access is gained and an
airway secured. The monitor shows VF and he is given a shock. Chest compressions are
continued and after he remains in VF. He is shocked for the second time and chest compressions
continue for two minutes. After this, the monitor is checked and remains in VF
According the current resuscitation guidelines, what is the immediate next management step?
(Please select 1 option)
A One shock and immediately restart chest compressions Correct
B One shock and check pulse
C Three shocks and check pulse
D Adrenaline
E Ten seconds of echocardiography

Current UK resuscitation guidelines emphasise the importance of minimising breaks in chest


compressions in a cardiac arrest situation.
After each shock chest compressions should be restarted immediately before anything else is
done. The rhythm assessment (and pulse check) should happen after two minutes of chest
compressions.
Also, current guidelines recommend single shock strategy.
Adrenaline is indicated after the third shock, and after chest compressions have recommenced.
The use of brief periods of echo (10 seconds) is now supported in an arrest situation (but should
be performed at the end of two minutes of compressions).
Therefore the most appropriate immediate step is A.
4. A 73-year-old man is brought to the emergency department in cardiac arrest. He was found by
his carer unconscious on the floor.
On arrival of the paramedics he was in VF and received two shocks prior to arriving at hospital.
He is intubated and has good IV access.
While in the emergency department he has had a further five shocks at the recommended energy
and the appropriate doses of adrenaline and amiodarone but despite this he remains in VF.
As part of the cardiac arrest team you are called to help. You look through his medications and
find he is on bendroflumethiazide.
Regarding the management of this patient's cardiac arrest, which is the next most appropriate
management option?
(Please select 1 option)
A Increase shock energy Incorrect answer selected
B Increase dose of adrenaline
C Lidocaine IV
D Magnesium sulphate IV This is the correct answer
E Repeat dose of amiodarone, at increased dose
Refractory VF is, of course, a worrying sign in a cardiac arrest situation.

It is worth considering other management options if repeated shocks are ineffective. Also it is
always worth repeatedly reviewing the "4 Hs and 4 Ts" for any possible reversible causes.
Magnesium sulphate IV is recommended for the treatment of refractory VF, if there is anything
to suggest the patient may be hypomagnesaemic (such as on medications which might cause this,
that is, thiazides).
There is no indication for increasing doses of adrenaline or amiodarone, or increasing shock
energy.
Amiodarone can be given again but this should be at the reduced dose of 150 mg.
Lidocaine is only recommended if amiodarone is unavailable, and/or has not already been given.
Therefore the most appropriate response here is D.
5. A 64-year-old man with a history of depression self presents to the emergency department four
hours after taking an overdose of his antihypertensive medication, which he thinks is a betablocker. He states he has been feeling light headed for a few hours but has not noted any other
symptoms.
His blood pressure is recorded as 96/60 mmHg and his pulse is 45. A 12 lead ECG reveals a
regular rhythm with normal QRS morphology. The PR interval is prolonged but there are no nonconducted P waves.
From the options below, which is the first step in this patient's management?
(Please select 1 option)
A Activated charcoal Incorrect answer selected
B Atropine This is the correct answer
C Glucagon
D Observation
E Temporal transvenous cardiac pacing
Overdose of beta-blockers or calcium channel blocker can lead to significant bradycardia.
If taken within one hour of presentation, activated charcoal should be tried.
If there is symptomatic bradycardia atropine should be used in the first instance.

Glucagon can be effective but this should be tried after atropine.


Pacing may be necessary if these drug treatments fail.
Therefore the correct answer here, in a patient who has symptomatic bradycardia more than one
hour after a probable beta-blocker overdose is B, atropine.
6, A 67-year-old lady is admitted, via the emergency department to the coronary care unit. She
complained of sudden onset palpitations and chest pain. On arrival in CCU she was in atrial
fibrillation. The on-call doctor prescribed intravenous amiodarone. This cardioverted to normal
sinus rhythm.
Three hours later the patient complains of some chest discomfort. Her blood pressure has
remained at 110/70 mmHg and heart rate increased to 190 The 12 lead ECG shows torsade de
pointes.
You call for immediate help from the cardiology registrar. She asks you to initiate which of the
following treatment options before she arrives to help?
(Please select 1 option)
A Amiodarone intravenously
B DC cardioversion
C Flecainide intravenously Incorrect answer selected
D Magnesium sulphate intravenously This is the correct answer
E Transcutaneous overdrive pacing
Drugs which prolong the QT interval (including amiodarone) can lead to torsade. This is a
malignant arrhythmia with a significant risk of deteriorating into ventricular fibrillation (VF) and
needs prompt treatment.
If any adverse features are present, synchronised DCCV is the treatment of choice. In this case,
the patient is stable and may need DCCV.
The initial management of torsade with no adverse features present is

Stop all drugs which prolong QT

Correct any electrolyte abnormalities and

Give IV magnesium (2 g IV over 10 minutes).

The patient may need transvenous overdrive pacing, but expert help is needed for this.
Therefore the correct answer is D.
7. This 52-year-old man was found collapsed on a golf course.
Cardiopulmonary resuscitation was initiated. On arrival of the paramedics, he was noted to be in
ventricular fibrillation.

What was the cause of his collapse?


(Please select 1 option)
Acute myocardial infarction

Incorrect answer selected

Acute rheumatic fever


Bacterial endocarditis
Cardiac Lyme disease
Lightning strike

This is the correct answer

The figure shows a 'ferning' or 'arborescent' rash pathognomonic of a lightning strike, also
known as Lichtenberg figures.
"The pathology of lightning, or keraunopathy, is known only to a few specialists." Space Science

8. A 75-year-old woman is admitted with a pre-syncopal episode.


She had felt as though she were going to 'black out' whilst out shopping and an ambulance was
called. She denies any history of chest pain or shortness of breath. She has a past history of
hypertension for which she has been taking a diuretic.
On examination she is conscious and has a BP of 85/50 mmHg. Her ECG is shown below.

Which of the following is the most likely diagnosis?


(Please select 1 option)
Atrial tachycardia with aberrant conduction Incorrect answer selected
SVT with aberrant conduction
Torsades de pointes
Ventricular fibrillation (VF)
Ventricular tachycardia This is the correct answer
The ECG shows a wide complex tachycardia with a rate of about 200 with marked left axis
deviation.
On careful examination of the rhythm strip there is evidence of independent atrial activity; P
waves can be seen 'marching through' the QRS complexes.

Even without demonstrating independent P waves, the QRS width, axis deviation and rate all
suggest a ventricular origin rather than a supraventricular origin.
This is not torsades as there is no characteristic twisting about the isoelectric line, and it is not
VF since there is a regular pattern to the QRS complexes.
Another possibility (not listed here) is ventricular flutter but this would normally have an even
higher rate.
9, A 63-year-old lady presented with sudden onset of central chest pain radiating to the left arm
associated with nausea and sweating. The pain had eased with sublingual nitrate spray given by
the ambulance crew, but was still present on arrival in hospital. She had a past history of
hypertension treated with bendroflumethiazide.
She had been admitted to hospital with chest pain two years previously, but was diagnosed as
having non-cardiac chest pain with a normal ECG and normal serum troponin-T. She was a
lifelong smoker of twenty cigarettes per day.
On examination, she appeared pale and sweaty. Her pulse was 100 beats per minute and regular
with a blood pressure of 130/85 mmHg. Heart sounds were normal and the chest was clear on
auscultation. A 12-lead ECG showed left bundle branch block (LBBB). The chest radiograph was
normal.
What treatment should this patient receive?
(Please select 1 option)
Aspirin, analgesia and intravenous nitrate Incorrect answer selected
Aspirin and clopidogrel
Aspirin and low molecular weight heparin
Temporary pacing wire
Thrombolysis plus aspirin and clopidogrel This is the correct answer
Thrombolysis with aspirin and clopidogrel is the best treatment for this lady acutely, as the new
onset of LBBB in the clinical context is highly suggestive of an acute myocardial infarction
(AMI).

This lady is not in complete heart block and is not haemodynamically compromised and
therefore does not require a temporary pacing wire.
The combination of aspirin and clopidogrel is indicated in AMI, but together with reperfusion
therapy, be it thrombolysis or primary angioplasty.
Two recent trials (Commit and Clarity) indicate the benefit of dual antiplatelet therapy.
Heparin and aspirin are not as effective as when a reperfusion therapy has been given.
Analgesia and intravenous beta-blockade have a place in the management of AMI but with some
form of reperfusion therapy.
10. A 70-year-old man who has diabetes and is a smoker has cholesterol of 6.7 mmol/l.
His ECG is unremarkable. He has a history of chest pain on exertion. His clinical examination is
unremarkable.
Which is the best test to order?
(Please select 1 option)
Cardiac MRI Incorrect answer selected
Coronary angiogram This is the correct answer
Myocardial perfusion scan
Stress echocardiogram
Transthoracic echocardiogram
The National Institute of Clinical Excellence has issued guidance as to the management of stable
chest pain.
There is enough evidence about this patient in terms of his age and number of risk factors to
justify going straight to an angiogram
11. An 80-year-old man presents to the emergency department with a two hour history of chest
pain.

He has an ECG that confirms an inferolateral myocardial infarction. His blood pressure is 80/60
mmHg and heart rate 110 bpm and regular. He has widespread inspiratory crepitations and his
heart sounds are difficult to hear but the casualty officer thinks there is a pansystolic murmur.
Which test is likely to confirm the diagnosis?
(Please select 1 option)
A. A 12 lead ECG to look for a right ventricular (RV) infarction Incorrect answer
selected
B. A BNP blood test to assess the degree of heart failure
C. An echocardiogram to assess the heart valves This is the correct answer
D. A stress echocardiogram to look for worsening ischaemia
E. A troponin blood test to assess the extent of infarction
The patient has a new murmur in the context of an acute infarction and is clinically in shock.
An echocardiogram is mandatory to exclude acute mitral regurgitation (option C).
An ECG would not exclude acute mitral regurgitation (option A); clinically there is a suggestion
of a new murmur.
Some credit could be given to considering an RV infarct however a standard 12 lead ECG is not
suitable for locating a RV infarct.
A BNP or troponin would not aid further management in this clinical scenario (options B and E).
A stress echocardiogram would not aid the diagnosis and should not be performed acutely
(option D).
12. A 47-year-old lady with a positive family history of hypertension and premature coronary
artery disease is referred to the outpatient clinic for assessment of poorly controlled
hypertension.
Her blood pressure in clinic is measured at 200/100 mmHg. An MRI scan of her aorta and renal
arteries shows severe atheromatous stenosis in both renal arteries.
What is best way of treating her elevated blood pressure?
(Please select 1 option)

ACE inhibitors

Incorrect answer selected

Alpha-blockers
Beta-blockers
Bilateral renal artery stenting

This is the correct answer

Methyldopa

The best long-term treatment for this lady is bilateral renal artery stenting.
Angiotensin converting enzyme (ACE) inhibitors are contraindicated.
Other pharmacological treatments are always going to come off second best to stenting, as
there will be untreated bilateral renal artery stenoses.

13. A 50-year-old gentleman presents to hospital 24 hours after suffering an episode of chest
pain. He has been pain free for the last 12 hours.
His electrocardiogram shows significant ST elevation in the anterior chest leads with Q waves
and loss of R wave progression.
What is the next stage in his management?
(Please select 1 option)
Anticoagulation with warfarin

Incorrect answer selected

Exercise test
Primary angioplasty
Thrombolysis
Transthoracic echocardiogram

This is the correct answer

The diagnosis is a late presenting anterior myocardial infarction (MI), now pain free. The next
stage in his management is to establish the extent of damage to his heart.
Thrombolysis and primary angioplasty are too late for myocardial salvage.
An exercise test is not the next step but would be later in the admission and warfarinisation may
not be the long-term treatment.

14. A 66-year-old gentleman with known ischaemic heart disease sustains a cardiac arrest at
home whilst watching television. His wife calls 999 and starts cardiorespiratory resuscitation.
On admission to hospital he is in sinus rhythm. A transthoracic echocardiogram reveals
significant left ventricular dysfunction.
What is the best long-term management strategy for this gentleman?
(Please select 1 option)
ACE inhibitor

Incorrect answer selected

Amiodarone
Beta-blocker
Implantable defibrillator

This is the correct answer

Mexiletine

This gentleman has ischaemic cardiomyopathy. He has presented with an out of hospital
cardiac arrest successfully resuscitated.
ICD implantation is the best long-term management strategy for this gentleman as there is a
clear morbidity and mortality benefit over and above anti-arrythmic therapy in this condition,
including amiodarone.
Long term mexiletine therapy has no place at this stage in his treatment and is probably proarrythmic in this situation.
Beta-blocker, statin and ACE inhibitor therapy all have a mortality benefit in either heart failure
treatment and ischaemia and may decrease the incidence of cardiac arrests, but regardless of
their usage ICD therapy is first line treatment in this patient. They are complementary therapies.

15. A 52-year-old gentleman with a known history of hypertension, attends outpatient clinic and
has a blood pressure of 150/90 mmHg.
He has previously reduced his salt intake but continues to drink six bottles of wine per week. He
is on a beta-blocker and thiazide diuretic treatment.
What is the next step in treatment?
(Please select 1 option)

Commence an ACE inhibitor

Incorrect answer selected

Increase thiazide diuretic dose to 5 mg


Reassurance
Reduction of alcohol intake

This is the correct answer

Reduction of salt in cooking ingredients

Reduction of alcohol intake is the next step in his treatment.


Non-pharmacological manoeuvres are paramount and first line in hypertension management
therefore an angiotensin-converting enzyme (ACE) inhibitor should not be the next step.
He has already reduced his salt intake.
Reassurance will not bring his BP below the current guidelines.
Increasing the diuretic dose gives very little further BP lowering effect but increases the side
effect profile.

16. An 81-year-old man is admitted to the Emergency Department after collapsing at home.
His wife calls an ambulance but he recovers spontaneously.
He had been admitted to hospital with chest pain three months previously and was diagnosed
as having an acute anterior myocardial infarction.
On examination he has a diffuse apex beat and a pulse of 82 beats per minute with a blood
pressure of 110/70 mmHg. His electrocardiogram (ECG) shows ST segment elevation across
the anterior chest leads and frequent ventricular extrasystoles.
What is the most likely diagnosis?
(Please select 1 option)
A. Acute anterior myocardial infarction (MI)

Incorrect answer selected

B. Heart failure with secondary ventricular tachycardia (VT)


C. Intermittent complete heart block post MI
D. LV aneurysm formation with secondary ventricular tachycardia

This is the correct answer

E. Sinoatrial node disease

The clinical presentation is suggestive of VT secondary to a specific cause. The persistant ST


elevation suggests the formation of a left ventricular (LV) aneurysm post anterior MI.
Heart block post anterior MI is most likely to occur within the first few days post MI rather than
three months.
B, D, and E are less likely due to the presenting symptoms and ECG findings.

17. A 64-year-old man is noted to have hypertension (180/100 mmHg). He has a history of a
myocardial infarction two years ago and has a background history of asthma.
What is the antihypertensive of choice for this patient?
(Please select 1 option)
Alpha-blocker

Incorrect answer selected

Amlodipine
Angiotensin converting enzyme (ACE) inhibitor

This is the correct answer

Beta-blocker
Thiazide diuretic

The antihypertensive medication of choice is an ACE inhibitor. This drug has clear morbidity and
mortality benefits in both patients with impaired and normal left ventricular function.
Beta-blockers are contraindicated with the asthma.
The other medications do not have the wealth of clinical benefits that the ACE inhibitors offer.

18. A 45-year-old lady is referred to the outpatient clinic by her general practitioner following the
finding of significant hypertension on a routine check-up.
Her blood pressure in clinic is measured at 180/100 mmHg. Her electrocardiogram reveals
changes characteristic of left ventricular hypertrophy.
What is the most likely cause for her hypertension?
(Please select 1 option)

Coarctation of the aorta

Incorrect answer selected

Conn's syndrome
Essential hypertension

This is the correct answer

Phaeochromocytoma
Renal artery stenosis

Essential hypertension is the most likely diagnosis as this is the most common cause of
hypertension.
However this lady clearly needs investigation for secondary causes of raised BP.

19. A 60-year-old man has been diagnosed with coronary artery disease.
Which of the following treatments do not decrease the incidence of coronary events?
(Please select 1 option)
Amlodipine

Correct

Clopidogrel
Gemfibrozil
Metoprolol
Ramipril

Calcium antagonists and nitrates do not prevent cardiovascular events occurring.


Antiplatelets, beta-blockers and angiotensin-converting enzyme inhibitors prevent
cardiovascular events.
Gemfibrozil, a fibrate, has also been shown to decrease the incidence of coronary events
20. You are called urgently to review a 54-year-old man who has developed acute
onset pulmonary oedema some 36 h after his myocardial infarction. On arrival you
note that his blood pressure is 95/50 mmHg with a pulse of 100/min regular and a
pan-systolic murmur is noted. There are crackles on auscultation of the chest
consistent with heart failure.

Which of the following represents the next investigation of choice in this


man?

Troponin I

Troponin T

Urgent chest X-ray

Referral for angiography

Urgent echocardiogram
E
The timing of this mans deterioration coupled with a murmur of mitral
regurgitation and acute pulmonary oedema suggests the onset of
papillary muscle dysfunction or even rupture. Echocardiogram is the
investigation of choice to demonstrate the mitral regurgitation.
Management involves the use of vasodilators such as sodium
nitroprusside and the use of angiotensin-converting enzyme (ACE)
inhibition if tolerated. Inotropic support with drugs such as dopamine or
dobutamine may also be required. The case should be discussed with
cardiothoracic surgical colleagues to assess suitability for surgical repair,
although this should be postponed until after haemodynamic stabilisation
if possible due to the high risks of peri-infarct surgical intervention.

You might also like